Đến nội dung

gadget nội dung

Có 169 mục bởi gadget (Tìm giới hạn từ 09-06-2020)



Sắp theo                Sắp xếp  

#178418 TWIn

Đã gửi bởi gadget on 01-02-2008 - 23:38 trong Quán nhạc

Mình là 1 fan hâm mộ của TWins ,mình đang tìm các bài hát hay của TWin ai biết bài nào hay post lên cho mình thửong thức.CẢM Ơn :D



#191208 Bài dễ

Đã gửi bởi gadget on 05-09-2008 - 09:06 trong Thi HSG cấp Tỉnh, Thành phố. Olympic 30-4. Đề thi và kiểm tra đội tuyển các cấp.

Cho $ f(x): [0,\infty) - > R$ thỏa mãn $f(x) = e^{\sqrt {x}} + e^{ - \sqrt {x}}$
Tính giới hạn của $\lim_{n \to \infty} \lim_{x \to 0, x > 0} f^{(n)}(x)$
Have fun :(



#78728 Chứng minh $n\le 2004$

Đã gửi bởi gadget on 16-05-2006 - 16:55 trong Tổ hợp và rời rạc

Cho tập S gồm n điểm trên mặt phẳng sao cho không có 8 điểm nào thẳng hàng và có không nhiều hơn 91 khoảng cách khác nhau nối các điểm thuộc S.Chứng minh $n\le 2004$




#178753 Số Fibo và Lucas

Đã gửi bởi gadget on 04-02-2008 - 14:49 trong Bất đẳng thức - Cực trị

Gọi $F_{n}$ và $L_{n}$ tương ứng là số Fibonacci và số Lucas thứ $n$
Cmr :$\forall n \geq 1$
$\dfrac{1}{2}(F_{n}^{\dfrac{1}{F_{n}}}+L_{n}^{\dfrac{1}{L_{n}}})\leq 2-\dfrac{F_{n+1}}{F_{2n}} $



#66872 tổ hợp

Đã gửi bởi gadget on 02-04-2006 - 17:14 trong Tổ hợp và rời rạc




#191170 Dãy tốt nhất

Đã gửi bởi gadget on 04-09-2008 - 11:50 trong Các dạng toán khác

Cho dãy $r_1 = 2$ và $r_n = \prod^{n - 1}_{k = 1} r_i + 1, n \geq 2$. Cmr trong tất cả các tập các số nguyên dương ${a_i}$ thỏa mãn :$\sum^{n}_{k = 1} \dfrac {1}{a_i} < 1,$ thì dãy $ r_1,r_2, ... , r_n$ là dãy có tổng nghịch đảo tiến tới gần 1 hơn cả.
Tức với mọi tập $a_{i}$ thì :
$\sum^{n}_{k = 1} \dfrac {1}{a_i} \leq \sum^{n}_{k = 1} \dfrac {1}{r_i} < 1.$



#68310 Tập số nguyên

Đã gửi bởi gadget on 09-04-2006 - 12:57 trong Tổ hợp và rời rạc

ta xây dựng bằng quy nạp theo n tồn tại dãy n số a_1...a_n thỏa mãn a_i-a_j|(a_i;a_j)
Giả sử đúng tới n=k,dãy a_1...a_k thỏa mãn
xét dãy ma_1...ma_k;m với (a_1-1)(a_2-1)...(a_k-1)|m



#191549 Một Bài Cũ

Đã gửi bởi gadget on 17-09-2008 - 16:25 trong Các dạng toán khác

Bài này có vẻ dễ xơi nhỉ
Có thể tổng quát với n=3k giác đều :lol:
Với k=1 ta được bài toán quen thuộc tam giác ABC đều với M 1 điểm thuộc cung Bc không chứa A thì MA=MC+mB
xuất phát từ bài toán với trường hợp đơn giản ta đi đến bài toán tổng quát
Giả sử $A_1,A_2,..A_3k $là 3k giác đều
Với $A_i$ bất kì tồn tại 1 cặp $A_j,A_k$ duy nhất sao cho $A_i A_jA_k$ là tam giác đều.
Sử dụng định lí Ploteme thì ta có tồn tại 1 đoạn bằng tổng 2 đoạn ( 3 đoạn $MA_i,MA_k,MA_j$ )
Từ đây ta sẽ xếp được các bộ 3 chia thành 2 nhóm thỏa mãn
chỗ này hơi khó giải thích :lol:
có thể tổng quát với các số $m>n;m+n=3k$ sao cho$ \dfrac{m}{n}\leq \2$



#191635 1 bài toán đẹp

Đã gửi bởi gadget on 19-09-2008 - 20:39 trong Các dạng toán khác

Bài này là đề Taiwan 2001 cũ mèm rồi :D
Dùng phản chứng nhé :
giả sử $(m,n)=1 \Rightarrow (5^m-1;5^n-1)=5^1-1=4 $:lol:
Biểu diễn$ 5^m-1=2^a.p_1^{m_1}...p_k^{m_k}$
từ đây ta có được $\phi(5^m-1)=5^n-1=2^{a-1}p_1^{m_1-1}(p_1-1)....p_k^{m_k-1}(p_k-1)$
Nên $m_i=1$
Đến đây bạn có thể chứng minh được rằng m bắt buộc phải lẻ.
Dễ chứng minh $ 5^m-1$ không thể có dạng lũy thừa của 2 được nên $k\leq1$(tức $5^m-1$ có ước nguyên tố lẻ )
Nếu m chẵn thì $5^m-1$ và $5^n-1$ đều chia hết cho 8 vô lí với :wub:
Với $m$ lẻ $m=2k+1$ ta sẽ có được a=2 .mặt khác do $5^m-1=5^{2k}.5 \equiv 1 (mod p_i)$ nên 5 là số chính phương $mod p_i$
dùng luật tương hỗ Gauss ta đi tới $p_i$ là số chính phương mod 5.
nên $p_i \in \{0;1;-1} (mod 5)$
$p_i$ không thể chia hết cho 5 được và nếu tồn tại i để $5|p_i-1$ thì $p_i-1|5^n-1 $ nên $5|5^n-1$ vô lí .
do đó $p_i \equiv -1 (mod 5)$
$ \Rightarrow 5^m-1 \equiv 4.(-1)^k \equiv (-1)^{k+1} (mod 5) \Rightarrow k$ chẵn.
Với k chẵn :
$5^n-1 =2.(p_1-1)..(p_k-1) \equiv 2(-2)^k $ do k chẵn nên $5^n-1$ đồng dư 2 hoặc -2 mod 5
Vô lí
Vậy ta có đpcm :lol:
Bài này công nhận hay thật :wub:
check hộ lời giải cho mình nhé.Lâu mới có thời gian làm toán thú vị như hôm nay.



#191895 Nhờ mọi người

Đã gửi bởi gadget on 05-10-2008 - 13:48 trong Các bài toán Lượng giác khác

Anh làm mẫu 1 bài thôi nhé vì các bài này đều phương pháp giống nhau,em cố gắng tự làm mới khá lên được
2.
Do $A+B+C=\pi $nên $sin(B+C)=sin A$
$sin(2kA) +sin(2kB)+sin(2kC)=sin(2kA)+2.sin[k(B+C)].cos[k(B-C)]=sin(2kA) +2(-1)^{k+1}.sinkA.cos[k(B-C)]$
(thay $ k(B-C)=k(\pi-(A+2C) )$ ta được )
$=2sin(kA)[cos(kA)-cos(k(A+2C)]=2sin(kA).2.sinKC.sin(k(A+C)]=4/(-1)^ksinKA.sinkB.sinKC$

Em chỉ cần chú ý sử dụng thành thạo các biến đổi với hàm sin và $cos$ khi có $\pi$ trong dấu ngoặc là được



#191728 Bài hay

Đã gửi bởi gadget on 29-09-2008 - 00:50 trong Bất đẳng thức và cực trị

Cho x,y là 2 số thực TM $x^2+y^2-xy=1$
Tìm min,max của $P= x^4+y^4-3xy$

Đặt $x^{2}+y^{2}=a;xy=b$ thì $a\geq 2b$
Từ giả thiết $a-b=1$ Nên$ b+1 \geq 2b \Rightarrow 1 \geq b$ :D
$P=a^{2}-2b^2-3b$
Thay $a=1+b $với điều kiện
biện luận theo tam thức bậc hai ẩn b là được :D hoặc nếu cần bạn có thể dùng phương pháp đồ thị khi tìm min .max tam thức bậc 2 này.



#179039 Học kỳ 1 talent HUT

Đã gửi bởi gadget on 07-02-2008 - 20:41 trong Đại số tuyến tính, Hình học giải tích

Không biết mình có làm sai chỗ nào không giải thấy có vẻ dễ dàng quá các bạn check hộ mình cái :)
Giả sử $dim(S)=k;$
Sử dụng cái $dim(Ker(f))+dim(Im(f))=n$
từ đây có được với mỗi $f\in Hom(V,R)$ đều có dim(Im(f))=n-k;
Giả sử $(S^{0})^{0}=W $từ đây dễ dáng thấy được $S\in W$
mà $dim(W)=k=dim(S)$
$S$ là không gian con của $W$ mà $dim(S)=dim(W)$ nên có được $S=W$(tính chất quen thuộc của kgvt)



#191168 Đơn giản và đẹp

Đã gửi bởi gadget on 04-09-2008 - 11:28 trong Các dạng toán khác

bài này bạn xét các trường hợp :
1. m có 2 ước nguyên tố cùng nhau là a,b
sử dụng tính chất hàm nhân tính của $\phi(m) =\phi(a)\phi(b)$
2.m có dạng $p^k$ chú ý là với p nguyên tố lẻ thì mọi $p^k$ đều có căn nguyên thủy nên chỉ phải xét trường hợp m$=2^k$



#65325 từ mathlinks

Đã gửi bởi gadget on 27-03-2006 - 14:03 trong Bất đẳng thức - Cực trị

Ta cần chứng minh
Cùng trừ 2 vế đi để viết về dạng tổng bình phương
Đến đây là dễ rồi quan trọng là ý tửơng



#83155 Họ tập hợp.

Đã gửi bởi gadget on 31-05-2006 - 10:24 trong Tổ hợp và rời rạc

bạn nào giải được bài này chưa



#80472 câu hỏi

Đã gửi bởi gadget on 22-05-2006 - 14:16 trong Tổ hợp và rời rạc

http://www.mathlinks...pic.php?t=15112 xem post 10 đây là lời giải của một học sinh bungaria và đã được giải đặc biệt cho lời giải độc đáo
Lời giải đó giống hệt lời giải của thầy thắng trong bài viết ''dùng cái ảo để đếm cái thật''...Chỉ khác là lời giải của thầy thắng là cho bài tổng quát nhưng từ lời giải này cũng không khó để suy ra lời giải bài tổng quát.Ý kiến của các bạn thế nào>mình luôn coi đây là 1 trong những bài viết hay nhất trên báo toán nhưng sau khi đọc cái này thì
:P :Rightarrow :Rightarrow



#85911 dãy số khó

Đã gửi bởi gadget on 11-06-2006 - 10:47 trong Số học

Chứng minh tồn tại một dãy số a_1...a_n thỏa mãn nếu ta thêm vào giữa http://dientuvietnam...mimetex.cgi?a_nhttp://dientuvietnam...tex.cgi?a_{n 1} một số các chữ số thì dãy số vô hạn nhận được không thể là phần lẻ của một số hữu tỉ
Chứng minh với điều kiện là không tồn tại dãy số nào như vậy



#85713 dãy toán số nguyên

Đã gửi bởi gadget on 10-06-2006 - 10:31 trong Số học

Day la bai thi putnam 1971(rat kho)
loi giai tren kalva minh doc khong hieu
http://www.kalva.dem...ln/psol716.html
ai giai duoc bai nay mịnh phuc sat dat



#68029 Bài số hấp dẫn

Đã gửi bởi gadget on 08-04-2006 - 10:25 trong Số học

lấy p nguyên tố cho trước
Xét dãy u_n=p^{2^n}+1.dễ thấy 2 số hạng bất kì nguyên tố cùng nhau
Vì vậy nếu đặt v_n là ước nguyên tố max của u_n thì v_n phân biệt dễ thấy v_2<p
Gọi m min sao cho v_m>p
->p(p^{2^m}-1)<p
->p(p^{2^m}-1)>p
Với mỗi p ta chọn được 1 số thỏa mãn ->có vô số số thỏa mãn



#192568 Tính giới hạn

Đã gửi bởi gadget on 20-10-2008 - 17:21 trong Các dạng toán khác

Cho $(u_n)$là một dãy số thỏa mãn:$u_n=\dfrac{1}{2}C_n^0-\dfrac{1}{4}C_n^1+\dfrac{1}{6}C_n^2-\dfrac{1}{8}C_n^3+...+\dfrac{(-1)^n}{2n+2}C_n^n $. với n=1,2,3....
Tinh $lim\dfrac{u_n}{u_{n+1}}$

Xét $ f_{n}(x)=C_n^0x-\dfrac{1}{2}C_n^1x^2+...+\dfrac{(-1)^n}{n+1}C_n^nx^{n+1}$
$f_n(x)'=(1-x)^n$ Từ đây : $f_n(x)=\dfrac{1}{n+1}-\dfrac{-1}{n+1}(1-x)^{n+1}$
Nên từ đây $u_n=\dfrac{1}{2(n+1)}$
Do đó $lim \dfrac{u_n}{u_{n+1}}=1$



#191875 Giải giúp em

Đã gửi bởi gadget on 04-10-2008 - 18:49 trong Bất đẳng thức và cực trị

Em bình phương 2 vế lên và sử dụng BDT Bunhia
$\sqrt{x+y+z}\sqrt{x}+\sqrt{3yz} \leq 2\sqrt{x+y}\sqrt{x+z}$
anh chỉ hướng dẫn thế thôi em tự làm tiếp đi nhé ! ngồi lo học đi đừng lo đập muỗi lắm :-?



#82761 Bài thầy Khoái

Đã gửi bởi gadget on 29-05-2006 - 19:10 trong Tổ hợp và rời rạc

Chứng minh tồn tại bảng 50.50 mà giá trị tuyệt đối tổng các số trên bảng <=50
xét 4 bảng 50.50 tạo thành bảng 100.100.Nếu không có bảng nào thỏa mãn đề bài thì tồn tại 2 bảng mà một bảng có trị tuyệt đối của tổng>50 và 1 bảng <-50
Xét phép tịnh tiến song song bảng 1 để nhận được bảng 2;mỗi lần bảng mới nhạn được có tổng các số thuộc bảng sai khác nhau<=2n từ đó có dpcm
Ý tưởng bài này giống bài IMO97



#191169 1 bài toán rất tuyệt vời

Đã gửi bởi gadget on 04-09-2008 - 11:30 trong Các dạng toán khác

Bài này là ISL 2005 đã được giải trên diễn đàn rồi bạn có thể search và tìm lại :(
nếu cần mình sẽ post lời giải hoàn chỉnh lên :D



#191709 Chinese Girls’ Mathematics Olympiad

Đã gửi bởi gadget on 28-09-2008 - 16:51 trong Tài nguyên Olympic toán

Cuốn này mình không có bản text ,mình thấy đề của cuộc thi này cũng bình thường mà :D bạn có thể vào mục tài nguyên (resource) của mathlinks click vào từng năm bên mỗi bài đều có línks solution đó bạn :D
Nếu bạn cần sách để in ra cho dễ học thì thử liên hệ anh Tuân :[email protected] thử xem nhé :(
Chúc bạn thành công.



#87307 bài khó

Đã gửi bởi gadget on 16-06-2006 - 17:17 trong Số học

Cho số nguyên tố http://dientuvietnam...etex.cgi?p=8k 7 http://dientuvietnam...tex.cgi?p|m^2-2
Chứng minh khi và chỉ khi lẻ